Just nu i M3-nätverket
Gå till innehåll

Derivering


Shaggo

Rekommendera Poster

Hej!

 

Egentligen är detta termodynamik, men det har med matematik att göra också.

 

Jag skall bestämma den kritiska punkten analytiskt för ett ämne.

 

Detta genom att ta Van der Waals tillståndslag och derivera den två gånger.

 

(p+a0/v²)(v-b0)=RT

 

Hur deriverar jag den? (dp/dv)T ?

 

Hur deriverar jag den andra gången?

 

Sedan skall man sätta båda till 0 ( (dp/dv)T = (d²p/dv²)T = 0 )

 

och lösa ut vad p (kritiska trycket) v (kritiska volymen) och T (kritiska temperaturen) blir enligt de uttrycken och Van der Waals lag.

 

Jag är helt borta. Jag förstår inte ens vad jag ska göra.

Mitt ämne är ammoniak som jag undersöker om det är till någon hjälp.

 

Länk till kommentar
Dela på andra webbplatser

Hej,

 

Lite ringrostig angående termodynamiken, men rent matematiskt borde följande fungera:

Du har ett samband med tre variabler (p, v, T) och tre konstanter (a0, b0, R).

- Lös ut p uttryckt i v och T.

- Derivera sedan uttrycket två gånger med avseende på v (dvs betrakta T som en konstant).

- När du sätter de deriverade uttrycken till noll får du två ekvationer med två obekanta (v, T) och kan (förhoppningsvis) lösa ut dessa i konstanterna.

- Sätt in dina nyfunna värden för v och T i uttrycket för p.

 

mvh

/Johan

 

[inlägget ändrat 2007-08-29 09:31:55 av Pejo]

Länk till kommentar
Dela på andra webbplatser

Ok.

 

Jag har löst ut p som:

 

p = (RT/V-b0) - (a0/v²)

 

Så nu ska den funktionen deriveras två gånger då. Jag måste nog leta efter min mattebok... Har glömt det där.

 

Länk till kommentar
Dela på andra webbplatser

Rest man Klaymen
p = (RT/V-b0) - (a0/v²)

Har du både V och v i uttrycket eller är det ett skrivfel?

 

Om det var ett skrivfel deriveras p med avseende på v till

p' = -(RT/v^2)+(2a0/v^3)

 

om jag inte gjort helt fel.

 

EDIT: glömde ett par minustecken, och att det skulle deriveras 2 ggr.

 

p'' = (2RT/v^3)-(6a0/v^4)

 

EDIT2: glömde att det inte alls ska vara minustecken :)

[inlägget ändrat 2007-08-29 10:52:30 av Rest man Klaymen]

[inlägget ändrat 2007-08-29 10:54:15 av Rest man Klaymen]

Länk till kommentar
Dela på andra webbplatser

Rest man Klaymen

b0 försvinner i deriveringen liksom alla andra konstanter.

 

R, T och a0 står kvar eftersom de står i uttryck tillsammans med v, exempelvis a0/v^2. Hade a0 stått ensamt hade det också försvunnit.

 

/ RMK

Jobbiga_färger.

 

Länk till kommentar
Dela på andra webbplatser

Hej igen,

 

Om jag läser ditt ursprungliga samband rätt:

(p+a0/v²)(v-b0)=RT

borde p bli

 

p = RT/(V-b0) - a0/V^2

 

och då blir

 

dp/dV = -RT/(V-b0)^2 + 2a0/V^3

 

d2p/dV2 = 2RT/(V-b0)^3 - 6a0/V^4

 

dvs lite snårigare ekvationer ...

 

mvh

/Johan

 

Länk till kommentar
Dela på andra webbplatser

Ja, det är snårigt.

 

Om man förutsätter att båda ekvationerna blir = 0 borde man väl kunna få ut ett samband mellan variablerna?

 

I den kritiska punkten är nämligen andraderivatan noll.

 

Man borde kunna skapa en ekvation som gäller för kritiska data där a och b är "borttrollade"?

 

 

 

Länk till kommentar
Dela på andra webbplatser

Hej!

 

Jaha, hur gör man det?

 

Nu vet jag inte hur mycket du kan om termodynamik, men jag ska försöka förklara vad det är jag vill uppnå.

 

För överkritiska medium skall man kunna skriva Van der Waals lag som en ekvation där molekylernas egenvolym (b - enhet (mol/m³)) och deras attraktionskrafter (a - enhet (N m^4/mol²) eller Pa m^6/mol²)) inte används. Dessa värden är experimentellt framtagna och unika för varje substans. För ämnen men en temperatur och i ett sådant tryck att det beskrivs som överkritiskt (ingen skillnad på västka och gas) så skall det finnas ett samband mellan T p och V. Om vi inför nya T p och V som T' = T/Tk (k för kritisk) p' = p/pk och V' = V/Vk i Van der Waals lag så skall man få fram ett sådant samband.

 

Jag är dock helt vilse i hur. Om någon kan svara så hoppas jag ni orkar skriva ut alla mellanled, så jag förstår vad som hänt.

 

Som ovan så vet jag att för den kritiska punkten (pk, Tk, Vk) så gäller:

 

(dp/dv)Tk = (d²p/dv²)Tk = 0

 

Så först tror jag att vi måste uttrycka a och b i Tk, Vk, och pk (vilka det nu blir) och sedan sätta in de värderna istället för a och b i ekvationen. Sedan borde det gå att förkorta och förenkla en del.

 

Länk till kommentar
Dela på andra webbplatser

Ok, jag tänkte inte så långt...

 

Lös ut p

 

p(v,T) = (RT - a/v + ab/(v^2))/(v-B) = (RTv^2 - a(v-B))/(v^2(v-B))

 

p(v,T) = RT/(v-B) - a/(v^2) efter förenkling

 

dp/dv = -RT/((v-B)^2) + 2a/(v^3) = 0

 

d2p/dv = 2RT/((v-B)^3) - 6a/(v^4) = 0

 

om du dividerar andra ekvationen med den första får du kvar

3/v = 2/(v-B) och får ut

v = 3b.

 

Lös ut T ur första derivatan och sätt in v = 3b, ger

T = 8a/(27Rb)

 

Sätt in v och T i ursprungsekv och få ut

p = a/(27b^2)

 

 

 

 

 

Länk till kommentar
Dela på andra webbplatser

Hej

 

Har en tjock gammal bok Textbook of Physical Chemistry, Classtone. Andra utgåvan, 1947. Van Nostrand förlag.

 

Där finns ett avsnitt Critical State, där man med van der Waals ekvation beräknar a och b, de du är ute efter. Har saxat 2 avsnitt av formlhanteringen, se bifogad bild och hoppas att de kan vara till någon nytta.

 

/Olle

 

[bild bifogad 2007-08-30 14:12:37 av Olle M]

976042_thumb.jpg

Länk till kommentar
Dela på andra webbplatser

Arkiverat

Det här ämnet är nu arkiverat och är stängt för ytterligare svar.

×
×
  • Skapa nytt...